« first day (28 days earlier)      last day (546 days later) » 
00:00 - 14:0014:00 - 00:00

12:02 AM
0
Q: Difference between the integral of floor and the triangular series

diligarI figured out that $$\int_0^x \floor t = x*\left(x-1\right)/2 when x=\floor x$$ But what would the equation look like when $x \neq \floor x$ ?

0
Q: Call/Put options for Finance

user3472798I'm working with put/call options for a finance class, and am having just a little bit of confusion with the formulae. For call options, I know that the formula to determine price (C(0)) is equivalent to C(t) = $\left(\frac{s^u - p_c}{s^u-s^d}\right)$ * s(t) - $s^d$ $\left(\frac{B(t)}{B(1)}\right...

0
Q: Help using density functions for ordered statistics

user99986Problem: Let Y denote the amount of milk (in gallons) remaining in a 1-gallon container on its expiration date. Suppose that the density function for Y is f(y)=2y for 0 \le y \le 1. Suppose that a random sample of 5 gallons of milk is taken and the amount of milk remaining on the expiration date ...

Welcome to Math.SE, user99986. Words such as help do not add information to titles. Please edit the title so that it better describes the specifics of your question. Do not hesitate to make it longer or include a formula if needed. This site uses MathJax formatting of formulas. More tips here. (from a bot)Normal Human 20 secs ago
0
Q: Boundary conditions on periodic Sturm Liouville Problem

user1620696I arrived at the following problem when using separation of variables to solve a PDE on $\mathbb{R}^2$ using polar coordinates. In that case I needed to impose the condition that $u(r,0)=u(r,2\pi)$ and $u_\theta(r,0)=u_\theta(r,2\pi)$ in order to have a continuous function. This led to the foll...

Title contains problem. Tagged pde, differential-equations. Boundary conditions on periodic Sturm Liouville Problem
0
Q: Factoring out an equation

jackI found this equation in a book (m0 × v0^2) + (m1 × v1^2) = (m0 × v0Final^2) + (m1 × v1Final^2) It says that Notice that you have a different equation with the same two unknown variables: v0Final and v1Final. You can now factor these out and come up with a single equation for each unkno...

Short title. Question contains step by step, please. Factoring out an equation
0
Q: Finding the moment of inertia for a sphere.

ICantCode4ShitI'm trying to find the moment of inertia for a sphere of radius 1, with density of $1-\rho^2$ at a point distance $\rho$ from the center. I already found the mass which is $8\pi/15$, but now I'm trying to find the moment of inertia if rotated around the z axis. Any help is appreciated, although...

Question contains step by step. Finding the moment of inertia for a sphere.
0
Q: Prove that for every n ∈ N is n^2 | (n + 1) ^ n -1

math16666666666Can anyone help me with this? Do anyone know the expansion of (n+1)^n but not using Taylor expansion?

0
Q: Is $f$ borel and/or lebesque measurable?

snowmanThe function $f: \mathbb R \rightarrow \mathbb R$ is defined by $$\begin{equation} f(x)=\begin{cases} 2^n, & x=n \in \mathbb Z \\ 0, & \text{otherwise} \end{cases} \end{equation}$$ Is $f$ borel measurable? Is $f$ Lebesque measurable? Please help on this. I want to understand why... Trying to se...

0
Q: Coverage of Fundamentals of Lines and Planes

FreshAirI was helping a high school student with some fundamental concepts of planes and lines, when I realized I am rusty on some definitions myself. I found some minimal coverage in his high school math and precalculus textbook, and I also found a section on my vector calculus text by Marsden. However,...

Tag (reference-request) should not be the only tag a question has. Please add a tag for a subject area to which the question belongs. (from a bot)Normal Human 20 secs ago
0
Q: Approximating trig functions through Taylor / Maclaurin

Zion ToddI have a test on Tuesday and I'm trying to review a section I wasn't here for during class and I'm really confused. I know how to find taylor and maclaurin polybomials but there is a question that asks me to approximate sin 4 degrees to five decimal places. I know I need to turn it into radia...

This site uses MathJax formatting of formulas. More tips here. (from a bot)Normal Human 20 secs ago
0
Q: determine of df of$X_1+X_2 $

B11b$X_1$ and $X_2 $are iid mixed type random variables with $f_{x_1}(0)=$ $f_{x_2}(0)=0.5$ and constant density it the interval (0,2] determine of df of$X_1+X_2 $ thanks

Short title. Short question. determine of df of$X_1+X_2 $
0
Q: Show that $\int_{-\pi}^{\pi}log|1-e^{i\theta}|d\theta=0$

user218512I am asked to show $\int_{-\pi}^{\pi}log|1-e^{i\theta}|d\theta=0$. So I start with noting that $\int_{-\pi}^{\pi}log|1-e^{i\theta}|d\theta=lim_{\epsilon->0} [\int_{-\pi}^{-\epsilon}log|1-e^{i\theta}|d\theta + \int_{\epsilon}^{\pi}log|1-e^{i\theta}|d\theta]=0$ So now I create a contour which is ...

0
Q: aggregate claim: $ F_s(20)$

B11bYou are given that $E[(S-30)_+]=8$ and $E[(S-20)_+]=12$ the only possible aggregate claim in (20,39] is 22 with $f_s(22)=0.1$ calculate$ F_s(20)$

Short title. Short question. aggregate claim: $ F_s(20)$
0
Q: Find the values of t for which the system has unique solution.

416256I have to find the values of t for which the system has a unique solution x+y+tz=1 x+ty+z=1 tx+y+z=-2 I'm not sure if I have to compute the determinant of the associate matrix and then find the roots. Would that be enough? Does the -2 en the third equation affects in any way?

0
Q: Combinations and Permutations of coin tossing.

TrK24Toss a fair coin 20 times. Find out how many outcomes have 7 H's and 13 T's.

This site uses MathJax formatting of formulas. More tips here. (from a bot)Normal Human 21 secs ago
0
Q: How to find the area of one loop of a lemniscate?

user294516http://i.imgur.com/vc9byVK.png Find a parametrization of the lemniscate (x^2+y^2)^2 = xy by using t = \frac yx as a parameter. Then use Area enclosed by C = \frac 12∮_C_2 (x dy - y dx)to find the area of one loop of the lemniscate.

Welcome to Math.SE, user294516. This site uses MathJax formatting of formulas. More tips here. (from a bot)Normal Human 21 secs ago
0
Q: sin(4x) = -2sin(2x) solutions in [0,2π)?

JohnMy textbook gives the following answer: sin(4x) + 2sin(2x) = 0 2sin(2x)cos(2x) + 2sin(2x) = 0 2sin2x (cos(2x) + 1) = 0 2sin2x = 0 sin2x = 0 2x = πk x = kπ/2 So in the interval [0,2π) you have the solutions 0,π/2,π and 3π/2. The book then shows the other solutions from (cos(2x...

This site uses MathJax formatting of formulas. More tips here. (from a bot)Normal Human 21 secs ago
0
Q: Need clarification on Linear Algebra textbook answer

Newb18I understand everything until the last part where they get 2/5 sqrt(5)??? i know the ||[2 -4]|| means sqrt(2^2 + -4^2) Photo of the question because i cant embed

This site uses MathJax formatting of formulas. More tips here. (autocomment)Normal Human 21 secs ago
0
Q: graph and kruskal clustering

lsfMy last problem is let U be a set of points from $R^3$ and d:RxR $\to$ $R_{\geq0}$ an euclidean distance. For every partition of U with k classes, ($S_1$,...$S_k$), we define a quality of it as the shortest distance between 2 points from different classes. The below algorithm determines the parti...

0
Q: How is this true? Mathematical induction proof

user293395so I have 1/2 + 1/4 + 1/8 + ... + 1/2^n = 1 - 2^(-n) to prove by mathematical induction. I did all the steps and I end up with: 1-2^(-n)+2^(-n-1)=1-2^(-n-1) I tried this out in Wolfram and another "calculator" and it says that it's true. How? How do I transform this so it's equal?

This site uses MathJax formatting of formulas. More tips here. (from a bot)Normal Human 21 secs ago
0
Q: $A$ is $\sigma$-algebra of subsets $\mathbb R$

snowmanLet $\Sigma$ be $\sigma$-algebra of subsets of $\mathbb R ^2$. Set $$A= \{ S \subseteq \mathbb R: S \times \mathbb R \in \Sigma \}$$ Prove that $A$ is $\sigma$-algebra of subsets $\mathbb R$. Please can someone show me how to do this.

0
Q: Prove $x_n$ converges IFF x_n is bounded and has at most one limit point

CraigI'm not entirely sure how to go about proving this so hopefully someone can point me in the right direction. The definition I have for a limit point is "$a$ will be a limit point if for a sequence $x_n$ there exists a subsequence $(x_{n_k})$ such that $\lim_{k\to\infty} (x_{n_k})=a$". In the fo...

0
Q: Help in computing complex singularites and residues

CervusI need help showing a couple of things. First one , how can I describe which kind of singularity (removable, essential,finite order pole) I have in the origin if the given function is $\frac{z^4}{(cotz-1)^2} $ since $(cotz-1)^2$ is zero when $z=0$ and obviously $z^4$=0 when $z=0$ seems like deriv...

0
Q: Rational Multivariable limit

Abhimanyu ChoudharyI am having some issues with the following multivariable limit: $$\lim_{x,y\to0,0} \frac{x^2+y^2}{x+y}$$ I am trying to show whether it exists and is equal to 0, or whether it does not exist. What I tried to do was convert it to polar coordinates and then show the limit was zero from there, how...

0
Q: Basic description of the ring $\mathbb{Z}[x]/(x)$

ErnI know that $\mathbb{Z}[x]$ is all the possible polynomials in $x$ with integer coefficients: $\mathbb{Z}[x]=\{a_0+a_1 x + a_2 x^2 + \cdots:a_n\in\mathbb{Z}\}$. Where $a_n$ are not all necessarily distinct. Modding out by $x$ basically cuts the polynomials off at the first power of $x$, as I u...

Title contains basic. [Basic description of the ring $\mathbb{Z}[x]/(x)$](math.stackexchange.com/q/1552423)
Words such as help are uninformative in titles. Please edit the title so that it better describes the specifics of your question. Do not hesitate to make it longer or include a formula if needed. More tips here. (autocomment)Normal Human 28 secs ago
0
Q: Convergence of a power series for all $x \in \mathbb{C}$

Jagdeep SinghI came across the following power series while looking at a problem. $$-1+a_1x+a_2x^2+a_3x^3+.......$$ where $a_0=-1$ and the choice of $a_1$ is arbitrary.The other coefficients are dependent on $a_1$ in the following way : $$a_n= (-1)^{(n+1)} \frac{n^{n-1}}{(n-1)!}.a_1^{n}$$ For $a_1=0$,such a...

Consider replacing (analysis) with a more specific tag for the relevant branch of analysis. (from a bot)Normal Human 21 secs ago
0
Q: Implicit differentiation to find derivatives of a function whose only defined by its derivative

user3258845A question I have asks if $K(x)$ satisfies $K(1)=0$ and $K'(x)={1\over{x}}$ then show: If $f(x)=K(10x)$ then $f'(x)={1\over{x}}$ So Im not sure if I have to prove it or do something else but this is what I did. $$ f'(x)=K(10x)^0\cdot K'(10x) \cdot 10 \\ f'(x)={1\over{10x}}\cdot 10={1\over{x}} $...

Questions tend to get more attention when they have a tag for a broad area of mathematics relevant to the question. Some of these tags might fit. (autocomment)Normal Human 21 secs ago
0
Q: prove that the finite group G has a fixed point in S if |G|=$p^r$, p prime, |S|=N, (p,N)=1

RobI am asked to solve the following Let G be a finite group with $p^r$ elements, where p is prime. if G acts on the finite set S with N elements and $(p,N)=1$, prove that there exists $s\in S$ such that $g*s=s,\forall g\in G$ I am able to get that $\sum_{g\in G}I(g)>0$ but I don't see how to conv...

0
Q: Can a module have an infinite number of compositions series?

dREaMIs ther an example of a module that has an infinite number of composition series? I would think not, if there is it would have to have an infinite number of submodules.

0
Q: Another polylog integral

David HIn the interest of housekeeping, I recently took a look at what what polylogarithm integrals are still in the unanswered questions list. Some of those questions have probably languished there because the solutions methods are presumably too tedious and too similar to previously answered questions...

0
Q: Probablity density functions X+Y and X/Y

melissaLet X and Y be independent random variables with X being uniformly distributed in [0, 1] and Y being exponential with parameter λ. How do I find the pdf of X+Y and X/Y?

 
1:25 AM
0
Q: Expected Value of edges

jbsdfI've been thinking about this for a while and I couldn't understand how we could apply this to a bigger case. There's a graph with x vertices and y edges. The vertices are randomly put into set A or set B, with a 1/2 chance for getting put in A, and a 1/2 chance for getting put in B. The cut is a...

0
Q: For $p>0$, does $\int_1^\infty x^{-p/x}$ diverge?

Jill_JohnsonFor $p>0$, does $\int_1^\infty x^{-p/x}$ diverge? I've tried the root test, the comparison test, and the limit comparison test without success. Any assistance would be appreciated.

0
Q: Hyptohesis testing query

Rajesh ThevarI came across this statement in hypothesis testing chapter and I am confusing myself how the answer came. 'What is the probability of finding 20 defectives out of 100 samples, if, in fact p=0.10? • The answer is ~0.002.' Can you please give me an idea how 0.002 came ? Using sampling distributio...

Welcome to Math.SE, Rajesh Thevar. Words such as query do not add information to titles. Please edit the title so that it better describes the specifics of your question. Do not hesitate to make it longer or include a formula if needed. More tips here. (from a bot)Normal Human 21 secs ago
0
Q: Find the series solution to y''+xy'+y=0 when x0=0. Show y1 and y2 are fundamental set of solutions.

SnazzySasquatchi have done some work to get the problem to look like this (not sure if its correct): 2a2 + a0 + Σ ((n+1)(n+2)a(n+2)+nan+an)xn=0 then making the coefficients =0 i get a2=-1/2a0 also getting an+2=-an/(n+2) Now i am not sure where to go from here....

 
0
Q: Answerer's answer locked for the chattiness of others

Iwillnotexist IdonotexistI have participated in a long comment chain under a specific answer to the HNQ question Why is my bare-metal 16x 2.93GHz cores computer performing poorer than a VPS with 4x 2.5GHz cores?. This answer was locked by @womble♦ with the message "This post has been locked due to the high amount of off...

 
Welcome to Math.SE, SnazzySasquatch. This site uses MathJax formatting of formulas. More tips here. (autocomment)Normal Human 21 secs ago
0
Q: What is the flaw in this induction proof?

PatExplain the flaw in the following induction argument which shows all of Lucas’ toys are the same colour. Proof: We will show by induction that: for every integer n ≥ 1, in any group of n of Lucas’ toys, all the toys in this group are the same colour. Basis: If Lucas had only one toy, then clearl...

Tagged proof-verification. What is the flaw in this induction proof?
0
Q: Best source on learning how to do limits

jb91I am just curious if there is some website or book comprehensively covering theme of counting limitis of functions.

0
Q: A question about event correlation.

TonyUnder a probability space, given a set of events $B_i, i\in I$ where $I$ is an index set. If $\forall J\subset I, k\notin J$ we have $\Bbb P(B_k|\bigcap_{j\in J}B_j) \ge \Bbb P(B_k)$, then we say the events $B_i, i\in I$ are positively correlated. The problem is Is $\Bbb P(B_k|\bigcap_{j\in J...

Words such as question do not add information to titles. Please edit the title so that it better describes the specifics of your question. Do not hesitate to make it longer or include a formula if needed. More tips here. (autocomment)Normal Human 21 secs ago
0
Q: Let $V$ be a finite-dimensional vector space over a field $F$, and $f ∈ L(V )$. Show that there is an invertible $g ∈ L(V )$ such that $gfgf = gf$

Squires McGeeNot sure where to start.. would someone please point me to the significance of invertability to the result of $gfgf = gf$?

0
Q: How to prove multiparameter onto functions?

smitty_werbermanjensenI know that to prove a single-parameter onto function, you just find x in terms of y. How do you do it when the function has parameters such as "f(r, s) = r + 3pi*s"?

0
Q: Solve a second order homogeneous ODE

EmadI tried to solve following second order ODE, But I am almost stuck : $$\frac{\partial ^2\sigma}{\partial x^2}-h(x) \sigma=0$$ where $h(x)$ define as: $$h(x)=G_o+\left(G_{\infty }-G_o\right)\left(1+\frac{2}{\pi }\sum _{m=1}^{\infty } e^{-\frac{\pi ^2 \mathcal{D} m^2 t}{L^2}}\frac {\cos (\pi m)-...

Tagged differential-equations but mentions "partial". Solve a second order homogeneous ODE
Tag (proof-explanation) should not be the only tag a question has. Please add a tag for a subject area to which the question belongs. (from a bot)Normal Human 21 secs ago
0
Q: Proof (a | b \land a\nmid c) \rightarrow a\nmid(b + c)

danny13Prove $\forall a\in \mathbb Z, \forall b\in \mathbb Z, \forall c\in \mathbb Z, (a | b \land a\nmid c) \rightarrow a\nmid(b + c)$. Maybe a gentle nudge in the right direction

Short question. Tagged proof-writing. Proof (a | b \land a\nmid c) \rightarrow a\nmid(b + c)
0
Q: Solving Nonlinear Differential Equation Problem

Disha WanMy Question First I let u = y' and employed the Chain Rule to obtain du/dx = du/dy * u But I am not sure where to go from there. Any tips, suggestions, or solutions to the problem would be much appreciated!

0
Q: Fast Fourier transform on 8 points

user294530describe the Fast Fourier Transition F(f) for a 1-periodic function f(t) given at eight points t=0,1/8,2/8,3/8,4/8,5/8,6/8,7/8.

Welcome to Math.SE, user294530. This site uses MathJax formatting of formulas. More tips here. (autocomment)Normal Human 21 secs ago
 
2:10 AM
0
Q: How to handle questions that require remedial and advanced help?

danhI'd like to know how others handle a question that asks about something too advanced for the questioner's apparent skill level. An answer to the explicitly-asked part of the question might help future readers, but the answer that will most help the questioner is probably more remedial, suggestin...

 
0
Q: Find the number of positive integers such that logarithm of whose reciprocals to the base 10 has the characteristic $-2$.

Vinod Kumar PuniaFind the number of positive integers such that logarithm of whose reciprocals to the base 10 has the characteristic $-2$. Let $x$ be a positive integer. Now the characteristic of $\log_{10}(\frac{1}{x})$ is $-2$ I dont know how to solve further.How to count number of positive integers?Please h...

0
Q: Finding representation of integers as 2-adic

Philip ParkHow do I find representation of -3, -1/5, and 1/5 as 2-adic integer? Is 2-adic representation same as binary expression: for example, 1/5 = .00110011... ?

0
Q: Prove $B_2= \sigma (R_2)$

cooldudemanLet $B_2$ be the $\sigma$-algebra of Borel subsets of $\mathbb R^2$, i.e. $B_2=\sigma (\mathbb G_2)$, where $\mathbb G_2$ is the collection of all open subsets of $\mathbb R^2$. Prove $B_2= \sigma (R_2)$ where $$R_2=\{ (a,b) \times (c,d): a<b \, \, \text{and} \, \, c<d\, \, \text{are real number...

Short title. Question contains please. Prove $B_2= \sigma (R_2)$
0
Q: including the matlab "engine.h"

VanGoHow do I include the matlab "engine.h" file in my C project (Visual Studio 2012) #include "engine.h" I've done what is shown above and added engine.h to source files, but when I try to compile I receive error "Cannot open include file: "engine.h"

0
Q: A question of the book Elements of the representation theory of associative algebras volume 1

pxsI'm reading the book "Elements of the representation theory of associative algebras volume 1".And I can't understand the proof of the proposition 3.11 on padge 124. The place where marked green "because P is injective,u factors through P",how to get this result?

0
Q: Let $p(t), q(t) ∈ C[t]$ be relatively prime, $A ∈ M_n(\mathbb{C})$. Show that $rank(p(A))+rank(q(A)) ≥ n$.

Squires McGeeLet $p(t), q(t) ∈ C[t]$ be relatively prime, $A ∈ M_n(\mathbb{C})$. Show that $rank(p(A))+rank(q(A)) ≥ n$. I have been stumped on this question for quite awhile. Could someone please enlighten me in regards to a fitting theorem? I'm assuming this is related to Bilinear and Quadratic forms but I ...

Question contains please. [Let $p(t), q(t) ∈ C[t]$ be relatively prime, $A ∈ M_n(\mathbb{C})$. Show that $rank(p(A))+rank(q(A)) ≥ n$.](math.stackexchange.com/q/1552529)
 
2:31 AM
0
Q: Prove that lim f(x)=L iff lim (x0+h)=L iff lim(f(x0+h)-L)=0

Sally I am completely lost. So do I prove the two directions (starting from the first one and starting from the last one)? and how would I prove this?

0
Q: Proving a set is Borel

cooldudemanProve that if $B \subseteq \mathbb R$ is a Borel set, then $B \times \mathbb R$ is a Borel in $\mathbb R^2$, i.e. $B \times \mathbb R \in B_2 $ I'm still trying to understand the definition properly. Please can someone show this. It would help me so much. Thanks in advance!

Short title. Question contains please. Proving a set is Borel
 
0
Q: Is using up-vote to disagree down-vote problematic?

GstestsoI know I can upvote any posts for any reason, but consider a situation: I see a zero score post, originally it neither needs to up vote or down vote. Suddenly it becomes negative score, I think the post should not be down voted, so I upvote the post. Or in simple, if I upvote the post, but is n...

 
0
Q: Little-o Question

SpawnKilleRI am trying to show that the following relation holds: \begin{equation} \log(1+ax) = log(x) + o(log(x)) \end{equation} as $x\rightarrow \infty$, where $a$ a positive number. I tried using Taylor expansion but I could not come to the results. Any hints would be really helpful! Thanks!

Words such as question do not add information to titles. Please edit the title so that it better describes the specifics of your question. Do not hesitate to make it longer or include a formula if needed. More tips here. (autocomment)Normal Human 20 secs ago
 
3:01 AM
0
Q: Should I have accepted a trivial add-homework-tag edit on this old question?

BrianI just rejected a suggested edit on this question, where someone had proposed adding homework but making no other changes. Since the question was two and a half years old, I rejected the edit with the 'causes harm' reason and the following detail: Adding 'homework' tag to this old question w...

 
0
Q: I make 25% of three pointers, if I shoots 10 of them, what is the probability that I will have made exactly 5.

Ilia LabkovskyI thought about doing (.25^5)+(.75^5), not sure if this indicates that he makes 5 in row and then misses 5 in a row or if it is correct.

0
Q: Prove H based on the definition provided

danny13H(n) = { \begin{array}{lr} 0 & n\leq 0\\ 1 & n = 1 \textrm{ or } n = 2\\ H(n-1) + H(n-2) - H(n-3) & n>2\\ \end{array} Prove that $\forall n\in \mathbb N, n\geq 1 \rightarrow H(2n) = H(2n-1) = n$. Maybe I'm just an idiot but I approached this question by drawing a graph. Assuming H= n. But thi...

0
Q: Prove that Petersen graph is class 2

LinProve that Petersen graph is class 2 ( class 2 = $\Delta(G) + 1$. I try to draw the petersen graph and i find only one method to coloring the outer cycle ( outer cycle is colored by $1,2,1,2,3$) and we know that the edge connecting the outer cycle with tne inner cycle. Because of that I need 4 ...

Title ends with a digit. Prove that Petersen graph is class 2
0
Q: Union of infinite sets

taninamdarLet $A = \cup_{i \ge 1} A_i$, $i = 1, 2, \cdots$. This is union of countably infinite sets. Also, $A_i \subsetneq B$ for all $i$, i.e. there exists at least one element in $B$ that is not in $A_i$ for every $A_i$. Then is it true that $A \subsetneq B$? Intuitively, it seems true because for eve...

0
Q: Fourier cosine series expansion of $f(x)=1$

MC989Fourier cosine series expansion of $f(x)=1 for 0<x<\pi$ Hint iss "thought is better than calculation".

0
Q: Expected payout of an optimal hand in Poker

Ali[enter image description here][1] I created a video poker hand evaluator and I am not getting the optimal hand and I know my mistake is because of the way I am calculating the expected payout. Above is a picture from the web version of the game, how this "2.510638" got calculated? This is a ja...

This site uses MathJax formatting of formulas. More tips here. (autocomment)Normal Human 21 secs ago
0
Q: Quotient Space Notation

HDMQuick question, mostly just for my knowledge, but I'm working on a problem: Determine whether the indicated set $A$ is an ideal in the indicated ring $R$: $A = \{0,2,4,6,8\}$ in $R = \mathbb{Z}/10\mathbb{Z}$. For short hand, can I denote $A$ as $2\mathbb{Z}/10\mathbb{Z}$?

0
Q: Proving that a function with multiple parameters is onto?

smitty_werbermanjensenHere's my situation: I understand that to prove the "onto" ness of a function it is necessary to show that X can be expressed in terms of Y by isolating it using standard mathematics. However, I am confused as to how one can prove the trait of being onto in a function that has multiple parameter...

0
Q: If |A| = |B| = 5, how many functions f: A → B are invertible?

RiGidIf |A| = |B| = 5, how many functions f: A → B are invertible? I'm a bit lost on how to start this problem, any help would be much appreciated.

0
Q: Coefficients of power series

user143763After expansion, we have $$ (x_1+x_2+\dots+x_n)^m=a_1x_1^m+a_2x_1^{m-1}x_2+\dots $$ where $x_{()}$ is the variable and constant indeces $n>m$. What is the expressions of all these possible coefficients $a_{()}$? Thanks in advance.

0
Q: We throw 2 dice, a Red and Yellow one.

Ilia LabkovskyA is the event that Red rolls 1, 2, or 3; B is the event that Red rolls 2, 4, or 6; and C is the event that the sum of the two rolls is 5. (a) Find p(A|B), p(B|C), and p(C|A) (b) Find p(A|B ∩ C), p(B|C ∩ A), p(C|A ∩ B) (c) Are the three events pairwise independent? Mutually independent? My at...

This site uses MathJax formatting of formulas. More tips here. (autocomment)Normal Human 21 secs ago
0
Q: Explaining the strict inequality with root x

MilosI drew the graph of root x but I cannot see the link to the strict inequality. Any hints would be welcome. enter image description here

0
Q: Prove that $k$ divides $a_k$

Jack FrostDefine the sequence $a_k$ recursively by $\displaystyle\sum_{d|k}a_d=2^k$ with $d>0$. Prove that $a_k$ is a multiple of $k$.

Short title. Short question. Prove that $k$ divides $a_k$
0
Q: Question on notation involving Divisors

Jake LebovicA partial quote from a book I am currently reading: "For example let X be a Riemann surface of genus 0 and $\mathbb{P}^1$ be the standard projective line. We choose a point $p \in X$. Because $\deg \left[ p \right] = 1 \geqslant 2g - 1$ ..." The question I have is with the notation $\left[ p \r...

Words such as question do not add information to titles. Please edit the title so that it better describes the specifics of your question. Do not hesitate to make it longer or include a formula if needed. More tips here. (autocomment)Normal Human 21 secs ago
0
Q: Equation of a Plane

Asuna HibariFind the equation of a plane perpendicular to each of the two planes x - y + z = 0 and 2x+y-4z-5=0 and containing the point (4, 0, -2) Need help! Thank you. ^_^

Welcome to Math.SE, Asuna Hibari. This site uses MathJax formatting of formulas. More tips here. (autocomment)Normal Human 21 secs ago
 
4:05 AM
0
Q: Font Size Discrepancy on Flag History

ᴉʞuǝWhen viewing the flag history page, question hyperlinks are appearing a bit larger than answer hyperlinks. About 4px larger I believe. A quick look at the console shows this CSS rule for question hyperlinks, but nothing setting the font-size and weight on answer-hyperlinks. .question-hyperli...

 
0
Q: Find the locus of the midpoint on ellipse

Jesse BilzerianFind the locus of the midpoint of the line joining the focus (ae,o) to any point on the ellipse x^2/a^2 + y^2/b^2 = 1

Welcome to Math.SE, Jesse Bilzerian. This site uses MathJax formatting of formulas. More tips here. (autocomment)Normal Human 21 secs ago
 
4:24 AM
0
Q: $\sum_{n=1}^{50}\arctan(\frac{2n}{n^4-n^2+1})$

Vinod Kumar PuniaFind the value of $\sum_{n=1}^{50}\arctan(\frac{2n}{n^4-n^2+1})$ $\sum_{n=1}^{50}\arctan(\frac{2n}{n^4-n^2+1})$ $\frac{2n}{n^4-n^2+1}=\frac{2n}{1-n^2(1-n^2)}$ I am not able to split it into sum or difference of two $\arctan$s.Please help me.

A title should not be all-MathJax; having some plain text helps with search and navigation. (autocomment)Normal Human 21 secs ago
0
Q: Recursively defining and proving through induction

daniel deathnew to the site, hopefully you guys will be able to help me. I need to recursively define asub(n) = 6asub(n-1)-9asub(n-2), and prove asub(n)=(5-n)*3^n Sorry for the weird format, I am not familiar with the format you use on here. I've already done the basis case for this induction problem, but...

Welcome to Math.SE, daniel death. This site uses MathJax formatting of formulas. More tips here. (from a bot)Normal Human 21 secs ago
 
4:39 AM
0
Q: Prove, $\sqrt{n} \le (n!)^\frac{1}{n} \le \frac{n+1}{2} $

Muhammad Abdullah GhaziProve the following inequality, $$\sqrt{n} \le (n!)^\frac{1}{n} \le \frac{n+1}{2} \ \ \ \ \forall \ n\in \mathbb{N} $$

0
Q: Prove that there exists no solution for $\sqrt{a}+\sqrt{b}=c$

0.5772156649...If $a, b$ are non-square whole numbers, and $c$ is an positive whole number, prove there exists no solution to the following equation: $$\sqrt{a}+\sqrt{b}=c$$

0
Q: The sum which gives $3^n$

AstromanSo I have the following which I must prove : $\sum_{(n1,n2,n3):n1+n2+n3=n)} \binom{n}{n1, n2, n3}$ = $3^n$ I'm not sure where I must begin. This is btw a multinomial.

Short title. Short question. Tagged proof-writing. The sum which gives $3^n$
0
Q: Solving complex integration 33

A.AWhile calculating electric field due to charged sphere i came across this integral Which i have no idea how to solve. Please help me out. $$\int\frac{(z-Ru)(du)}{{(R^2+z^2-2Rzu)}^{3/2}}$$ It is given as hint that it can be solved by partial fractions.

Question contains please. Solving complex integration 33
0
Q: The sequence $x_{n}$=$cos(x_{n-1})$ is convergent or not, if $x_{0}$=0.

HemantI am having trouble in handling such sequences and don't know how to proceed. Please help me. Thanks in advance.

0
Q: Umbrella Markov chain problem

Robert SmithA man has an umbrella, and he commutes from his house to work and back. If it is raining, and he has an umbrella, he takes his umbrella. If it is not raining, or he does not have an umbrella, he does not take it. I am trying to establish the transition probabilities for the Markov chain. I unders...

Short title. Title contains problem. Umbrella Markov chain problem
0
Q: Getting the derivative of the inverse of a function

hmirGiven $f(x)$, how would I find $(f^{-1})'(x)$? As an example how would I find that for this problem: $f(x) = 4x^3 + 5x + 2$

 
5:19 AM
-1
Q: Is stackoverflow now the C/C++ student homework helpdesk

markshancockThe past few days I decided to spend some time on stackoverflow and answer questions. I have mostly been watching mostly C and C++ questions. It seems like 90% of the questions I am seeing are students asking for help with their programming homework. Most are really basic. Is this the norm now.

 
5:33 AM
0
Q: Generalized Harmonic numbers

msmithsonI'd like to be able to prove the following inequality: $\frac{{{H_{n, - r}}}}{{{n^r}\left( {n + 1} \right)}} \le \frac{{{H_{n - 1, - r}}}}{{n{{\left( {n - 1} \right)}^r}}}$. It's clear that as $n \to \infty$ we get equality, the limit on each side is $1/(r+1)$, and it also seems clear that this ...

0
Q: $c$ is the value of $x^3+3x-14$ where $x=\sqrt[3]{7+5\sqrt2}-\frac{1}{\sqrt[3]{7+5\sqrt2}}$.Find the value of $a+b+c$

Vinod Kumar Punia$a=\sqrt{57+40\sqrt2}-\sqrt{57-40\sqrt2}$ and $b=\sqrt{25^{\frac{1}{\log_85}}+49^{\frac{1}{\log_67}}}$ and $c$ is the value of $x^3+3x-14$ where $x=\sqrt[3]{7+5\sqrt2}-\frac{1}{\sqrt[3]{7+5\sqrt2}}$.Find the value of $a+b+c$. I tried to solve and simplify this problem but no luck.Please help m...

Question contains please. [$c$ is the value of $x^3+3x-14$ where $x=\sqrt[3]{7+5\sqrt2}-\frac{1}{\sqrt[3]{7+5\sqrt2}}$.Find the value of $a+b+c$](math.stackexchange.com/q/1552710)
0
Q: About 2-adic representation of integers

Philip ParkHow would I express -3 in 2-adic representation? Is it just revercimal calculation of binary expression of -3? like: -3 = -11 in binary, so using revercimal, -11. in binary = 01. ?

0
Q: Natural Deduction Help

ellieI desperately need help on these three natural deduction problems. If anyone can help, it'd be greatly appreciated.

Welcome to Math.SE, ellie. Words such as help do not add information to titles. Please edit the title so that it better describes the specifics of your question. Do not hesitate to make it longer or include a formula if needed. More tips here. (from a bot)Normal Human 20 secs ago
 
5:54 AM
0
Q: Grade 10 system of linear equation problem

cvghstryhA farmer harvested $1$ section (which is $640\, acres$) of wheat and $2$ sections of barley. The total yield of grain for both areas was $99,840\, bushels$. The wheat sold for $6.35\, /bushel$ The barley sold for $2.70\, / bushel$. The farmer received $363,008$ for both crops. What was the yiel...

0
Q: Any matrix can be reduced to a "special" matrix by elementary operations

LewisDefinition. Let $A$ be an $r \times s$ integer valued matrix. $A$ is "special" if there exists an integer $k$ such that $a_{ij}=0$ unless $i=j$ and $i \leq k$ and $a_{ij} \neq 0$ if $i=j \leq k$. Show that any matrix $A$ can be reduced to a "special" matrix by elementary row and column operat...

0
Q: What does "The value that the sampling distribution of the estimator be will be centered at" mean?

user3315748"The value that the sampling distribution of the estimator be will be centered at"

0
Q: What's "ancient time"?

mavaviljI've found a reference to "ancient time" from Google. It's mentioned in e.g. the book Sobolev Inequalities, Heat Kernels under Ricci Flow, and the Poincare Conjecture, by Qi S. Zhang. What's "ancient time" and why is the name such?

Short title. Short question. What's "ancient time"?
0
Q: Analysis self study

showdateI'm an engineering student who is doing some self study in analysis. The book I'm using is Maxwell Rosenlicht's "Introduction to Analysis". My background includes multiple variable calculus, ODEs, and some linear algebra (not proof based). As you likely know, all the exercises in the book involve...

0
Q: Winning Strategy with Addition to X=0

Analysis15Problem: Two players play the following game. Initially, X=0. The players take turns adding any number between 1 and 10 (inclusive) to X. The game ends when X reaches 100. The player who reaches 100 wins. Find a winning strategy for one of the players. This is my solution, which hopefully you c...

Title ends with a digit. Winning Strategy with Addition to X=0
Consider replacing (analysis) with a more specific tag for the relevant branch of analysis. (autocomment)Normal Human 20 secs ago
 
6:13 AM
0
Q: $\tan\frac{\pi}{16}+\tan\frac{5\pi}{16}+\tan\frac{9\pi}{16}+\tan\frac{13\pi}{16}$

Vinod Kumar PuniaFind the value of the expression $\tan\frac{\pi}{16}+\tan\frac{5\pi}{16}+\tan\frac{9\pi}{16}+\tan\frac{13\pi}{16}$ I identified that $\frac{\pi}{16}+\frac{13\pi}{16}=\frac{5\pi}{16}+\frac{9\pi}{16}=\frac{14\pi}{16}$ $\tan(\frac{\pi}{16}+\frac{13\pi}{16})=\tan(\frac{5\pi}{16}+\frac{9\pi}{16})$ ...

A title should not be all-MathJax; having some plain text helps with search and navigation. (autocomment)Normal Human 20 secs ago
 
0
Q: Should i delete the answer where OP gives own answer from my logic

ketanI had answered This Question. Owner of the question understood my answer properly and then they apply given logic and answer to their own question. Now, should i delete my answer? Or keep it there which helps some one else?

 
0
Q: A question about proving continuity using an epsilon-delta proof

PaninininiI have to prove that the function $f\colon [0,\infty)\to R$, defined by $f(x)=\lbrack\!\lbrack \sqrt{x}\rbrack\!\rbrack$ $\forall$ $x\in [0,\infty)$, is continuous at $c=2$. The function $f(x)=\lbrack\!\lbrack \sqrt{x}\rbrack\!\rbrack$ takes a number and rounds it down to the closest integer if a...

0
Q: How to prove these basic concepts about integration?

Sayantan SantraWhile we were introduced to integration, we were told about some basic concepts that, as we were told, could not be proved based on our level of sophistication. They are as follows: $\int_a^b \! f(x) \, \mathrm{d}x=\phi(b)-\phi(a)$, where $\phi$ is a primitive of $f$ in $[a,b]$ $\int_b^a \! f(x...

 
6:48 AM
0
Q: Trying to plot the partial sums of a series. Matlab.

M4thGyI want to plot the partial sums of the reciprocals of the squares between 1 and 10. So far, I've got the following code. It's something along these lines but I can't quite manage it. I need to sum the terms so far for each point between 1 and 10 but all I know how to do is sum all the terms. f...

0
Q: Logical Expression

WongsakornI saw this in the text. F = ~D~H~L + D~H~L + D~HL and it said this is equivalent to F = D~H + ~D~H~L Anyone know what is the process to this equivalence. I try to solved it but I'm not able to.

Short title. Logical Expression
 
7:09 AM
0
Q: How to prove $\tanh ^{-1} (\sin \theta)=\cosh^{-1} (\sec \theta)$

Vinay5forPrimeAs the question says How to prove $$\tanh ^{-1} (\sin \theta)=\cosh^{-1} (\sec \theta)$$ I have tried to solve it The end result that got for LHS $$=\log \frac{1+\tan\frac{\theta}{2}}{1-\tan \frac{\theta}{2}}$$ I am stuck here Please help

0
Q: Alternative proofs (algebra)

HawkI wrote some short alternative proofs (sketches mostly) to my book, can someone tell me if they are okay. The unity of a subfield is the unity of the whole field. Let $H \subset F$ with $F$ being the field. Let $1_H=hh^{-1}$ for every $h$, and let $1_F = ff^{-1}$ for all $f \in F$. Let $f =...

 
0
Q: Adding a Reference List to the MathOverflow Posts

Seyed TorbaghanThere are two very useful link lists in the right side of each MathOverflow post. One of them shows the related posts and the second one is for the other MathOverflow posts that are linked to that particular post somewhere in the question, answers or comments. But there is no list for the exter...

 
7:25 AM
0
Q: Meromorphic Function on Extended Plane

Marco ArmentaHow do I prove that every meromorphic function on the extended plane is a rational function?

0
Q: Show that trace($\gamma$)=trace($\gamma^{-1}$)

Tony Let $\gamma:[0,1]\to \Bbb C$ be a closed rectifiable curve and consider $\gamma^{-1}:[0,1]\to \Bbb C$ given by $\gamma^{-1}=\gamma(1-t)$. Show that trace($\gamma$)=trace($\gamma^{-1}$). I tried to show that $\gamma$ and $\gamma^{-1}$ is equivalent, since equivalent paths has the same trace. ...

0
Q: Help resolving this integral.

TBBTForgive me for asking this question. I am deriving the Wigner function, $$ W\left(x_{1},p_{1},x_{2},p_{2}\right)=\frac{1}{4\pi^{2}}\int dx'_{1}dx'_{2}e^{-ip_{1}x'_{1}-ip_{2}x'_{2}}\Psi\left(x_{1}+\frac{x'_{1}}{2},x_{2}+\frac{x'_{2}}{2}\right)\Psi^{\ast}\left(x_{1}-\frac{x'_{1}}{2},x_{2}-\frac{x'_...

Words such as help are uninformative in titles. Please edit the title so that it better describes the specifics of your question. Do not hesitate to make it longer or include a formula if needed. More tips here. (autocomment)Normal Human 21 secs ago
0
Q: How many binary sequences are there which have given numbers of substrings 00, 01, 10, 11?

kazukiMy teacher told me to write a program to solve this problem, but I thought it would be more fun to solve this without brute force of my PC. To explain this problem, for instance, 0000001001101111 has six 00s, three 01s, two 10s, four 11s.I tried to solve this by using recurrences.Define the numbe...

Consider adding a tag for a broader subject area to which the question belongs. Some of these tags might fit. (autocomment)Normal Human 21 secs ago
 
7:48 AM
0
Q: Permutation of 10 numbers, position 1 or position 2 not allowed

aprilduckWe have a permutation of $\pi$ numbers $\{1,2,...,10\}$. Let $A_1$ be $\pi(1)>1$ and $A_2$ be $\pi(1)>2$. (number on position 1 or 2 must be greater thatn 1 or 2, respectively). What is the probability of $A_1$ and $A_2$? Are $A_1$ and $A_2$ independent? I don't know how to approach this problem...

 
8:07 AM
0
Q: Differential equation problem, unit-step function

Linear Algebra ScrubSuppose we model a physical phenomenon with a 2nd order linear differential equation: $a_2$(t)y''+$a_1$(t)y'+$a_0$(t)y=$f(t)$, where 't' stands for time. In choosing an appropriate driving function f(t), suppose we want just the segment of a certain function $g(t)$ for 5≤t<10 only. Further, suppo...

0
Q: Beginners Question about the convolution theorem

user1862770I have a formular for the convolution theorem, and read several chapters in several scripts about it. This is the formula: $(f*g)(x)=\int_{\mathbb{R}^d}f(x-y)g(y)dy$ However much I read, I cannot figure out where exactly the y comes from. So, if I had an excercise where I have to convolute to ...

Welcome to Math.SE, user1862770. Words such as question are uninformative in titles. Please edit the title so that it better describes the specifics of your question. Do not hesitate to make it longer or include a formula if needed. More tips here. (from a bot)Normal Human 21 secs ago
0
Q: Prove that $2P$ is a flex of $C$ collinear with $P$ and $3P$ and $4P$ is a flex of $C$ collinear with $5P$ and $3P$.

Peachy ChiuI encountered this problem from Conics and Cubics by Bix. Please help me answer this. Let $C$ be a nonsingular, irreducible cubic with a flex $O$. Add points (commutative) of $C$ with respect to $O$ as base point. Let $P$ be a point on $C$ of order 6. Prove that $2P$ is a flex of $C$ collinear w...

0
Q: PatrickJMT posted these questions as review for precalc - are they enough?

idigr234https://drive.google.com/file/d/0B1YZD9uzvB5TVXdneEcyeVhsM3c/edit I can do most of them fine, but I don't feel ready for calculus. I'd love to start learning calculus but I'm not sure if I am ready and want a second opinion to see if these are enough for me to start calculus. I am skeptical beca...

Welcome to Math.SE, idigr234. Tag (reference-request) should not be the only tag a question has. Please add a tag for a subject area to which the question belongs. (autocomment)Normal Human 21 secs ago
0
Q: Menger Theorem, bounded length

FaneSpoitoruA straightforward corollary to Menger's Theorem states that if we pick two non-adjacent u,v∈V then the maximum number of internal vertex-disjoint u−v paths is equal to the minimum size of an u−v vertex-cut. Let's put in place a bound for the length these path have. Denote A_n(u,v) the maximum nu...

This site uses MathJax formatting of formulas. More tips here. (from a bot)Normal Human 21 secs ago
0
Q: Taking numbers away and then add the remaining to get 100

anonymousThis question came up in a math competition a few weeks ago. My reasoning for (a) was that if we took away the 9 smallest numbers (1-9), the smallest 9 numbers that we would then be able to choose from then would be 10, 11, 12, 13, 14, 15, 16, 17, 18. The sum of these numbers is already more ...

Tag (contest-math) should not be the only tag a question has. Please add a tag for a subject area to which the question belongs. (from a bot)Normal Human 21 secs ago
 
8:30 AM
2
Q: My code was edited to different indentation / code styling, I wish to revert it, What's the best course of action?

Michael DibbetsIf you look at the following review http://stackoverflow.com/review/suggested-edits/10368531 you see that my code was edited to suit someone's specific flavor of identation and code styling. Now I disagree with this specific flavor of identation, but worst of all, it doesn't "Fix" anything in m...

 
0
Q: Help with Claims about formal languages

Yinon ElirazGiven an alphabet $\Sigma$ and 3 words $u,v,w \in \Sigma^*$, and 3 claims about $u,v,w$: 1. $w^5=v^3$ 2. $uvw=wuv$ 3. $wu=uw$ Which claim implies the other and which not? When I first saw the claims, it seems like $2\implies 3$ and $not(1\implies 3)$. That is because intuitively, if we remove $v$...

Words such as help do not add information to titles. Please edit the title so that it better describes the specifics of your question. Do not hesitate to make it longer or include a formula if needed. More tips here. (from a bot)Normal Human 21 secs ago
 
0
Q: Follow-up reminder option for posts

Peter DunihoVery rarely, I run into a situation where Stack Overflow essentially says to me "can't do that now, come back later". For example, I've already voted on a post, and wish to take advantage of my opportunity to vote again after the required time has passed. Or I would like to check in on a post af...

 
0
Q: Decreasing eccentricity

SashaLet us consider surfaces $P_t = \partial\{v<t\} $ moving under inverse mean curvature flow where $v$ is a function defined on $\mathbb{R}^n \setminus \{0\}$. I need to show that the eccentricity $ \Theta(P_t)$ is decreasing. I want to use expanding spheres as barriers and I think that since w...

0
Q: Question related to Boolean Algebra.

KavitaI am asked to prove that order of a Boolean Algebra cannot be prime greater than 2. I have a dificulty to show this in an appriopriate way. I know the definition of Boolean Algebra. The definition I have seen is as follows: A structure (B,meet,join) is called a Boolean Algebra if B is distribu...

Words such as question do not add information to titles. Please edit the title so that it better describes the specifics of your question. Do not hesitate to make it longer or include a formula if needed. More tips here. (autocomment)Normal Human 21 secs ago
 
9:00 AM
3
Q: many answers in android-camera tag are spreading a lie

Alex CohnSorry for technical details, the question is below. Many posts tagged android-camera spread a wrong pattern: quite a few upvoted or accepted answers recommend to use Camera.open(CameraInfo.CAMERA_FACING_FRONT) instead of enumerating cameras with for (int camIdx = 0; camIdx < Camera.getNumberOfCa...

 
0
Q: Solvability of Heisenberg group

MamboIs Heisenberg group solvable? Does solvability depends on the topology?

 
9:23 AM
0
Q: Extreme Values of Functions

Akshar GandhiFind the solution : sin 5theta + cos 4theta=2 I am preparing for IIT and just stumbled upon this question.

Short title. Short question. Extreme Values of Functions
 
9:46 AM
0
Q: Failed close audit and I don't understand why

ReenoI just failed this audit. Here's the question. Frankly I don't understand why this should be a good question. It's basically a short description of the problem with "give me teh codez". This could be closed as too broad or off-topic -> mcve missing but I really don't think it's a good question an...

 
0
Q: Classification of manifold of dimension 1

RickBy my course, all manifold of dimension 1 is isomorphic to $(0,1), (0,1],[0,1)$ or $\mathbb S^1=\{x^2+y^2=1\mid x,y\in\mathbb R\}$. I was thinking of a curve in the plan, with a knot. (See picture) I agree that the gluing (in pink) is possible in $\mathbb R^3$, but if we consider this curve in ...

0
Q: Solving recursions with max

gusThe question is general, but I'll first give a simple example. Suppose you have a candy machine with $N$ candies. The machine is weird, when you give it a quarter it gives you $1$ to $N$ candies (all numbers equally probable), this is called one "buy". You may assume you have more than $N$ quart...

0
Q: Is there a proven way to calculate the entry point(first occurence) of a factor m, in the Fibonacci sequence?

Robert WhiteI saw a comment at the OEIS website for the sequence of entry points, of Fibonacci factors. https://oeis.org/A001177 It referenced a paper by Mark Renault in 1996, with the quote from OEIS: http://webspace.ship.edu/msrenault/fibonacci/FibThesis.pdf If m has prime factorization m=p1^e1 * p2^e2 *...

This site uses MathJax formatting of formulas. More tips here. (from a bot)Normal Human 23 secs ago
0
Q: Permutation and combination 3727727

Samagrahttp://s15.postimg.org/kjkxd1nxn/Screenshot_2015_11_29_23_08_38.jpg Ques no. 1 &2 . It is very interesting.

0
Q: norm of non-square matrix

123I have the question about the non-square matrix. If A is m by n matrix, the m is not equal n. x is a vector which the dimension is n by 1. Is it correct that ||Ax|| <= ||A|| ||x|| If A is m by n matrix and B is n by m matrix, is it correct that ||AB|| <= ||A|| ||B||

0
Q: Bounded analytic functon with small derivatives

user244974A question from the theory of bounded analytic functions. Let $f$ be analytic in the circle $D: |z|<1$ and bounded in $D$ by absolute value by a constant $M>0$. We assume that $N$ derivatives of $f$ at $z=0$ are bounded by absolute values by known constants $\epsilon_i$, $i=0\dots N$ (small one...

Consider adding a tag for a broader subject area to which the question belongs. Some of these tags might fit. (from a bot)Normal Human 31 secs ago
0
Q: How to create a bijection between (0,1) and (0, ∞)?

Jesper MagnussonI don't understand how to do this. The tip I have for the question is to first find a bijection between (0,1) and (1,∞).

0
Q: Solve the equation,$\sqrt{\log(-x)}=\log{\sqrt{x^2}}$

diyaSolve the equation,$\sqrt{\log(-x)}=\log{\sqrt{x^2}}$ $\sqrt{\log(-x)}=\log{\sqrt{x^2}}$ $\sqrt{\log(-x)}=\log{|x|}$ Now two cases arise,when $x>0$ and when $x<0$ When $x<0$, $\sqrt{\log(-x)}=\log(-x)$ I found $x=-1,-10$ When $x>0$ $\sqrt{\log(-x)}=\log x$ $\log(-x)=\log x\times\log x$ I could...

Short title. Question contains please. Solve the equation,$\sqrt{\log(-x)}=\log{\sqrt{x^2}}$
0
Q: Basic probability…balls extraction

oso_togariim stuck with a basic probability problem: n balls --> n-1 extraction. Only one black ball. No replacement. example: 7 balls(6 white, 1 black). 6 extractions. i know that the probablity of 6 whites is: 6/7 · 5/6 · 4/5 · 3/4 · 2/3 · 1/2 = 0.14 aprox, so the prob of get the black ball is: 1 - 0...

Welcome to Math.SE, oso_togari. This site uses MathJax formatting of formulas. More tips here. (autocomment)Normal Human 25 secs ago
0
Q: Laplace transform question

Linear Algebra Scrub$f(t)$=$g$$(t-10)$$U$$(t-15)$$-$$g$$(t-10)$$U$$(t-20)$ The above $f(t)$ contains terms of the form $f$$(t-a)$$U$$(t-b)$, where $a$ doesn't equal $b$. Describe the form that $L${$f$$(t-a)$$U$$(t-b)$} takes. [Hint: The formula for L{g(t)U(t-a)}$=$$e$^${-as}$L{g(t+a)}

Words such as question are uninformative in titles. Please edit the title so that it better describes the specifics of your question. Do not hesitate to make it longer or include a formula if needed. More tips here. (from a bot)Normal Human 25 secs ago
0
Q: The exterior derivate

MarkI am trying to show that $d(a \wedge da)=0$ if $k$, the degree of k-form $a$ is even. I have said: $=da \wedge da + (-1)^k a \wedge d^2a$ I believe the first term is zero due to repeated indices and the second is zero since $d^2=0$. However, I need it not to always be zero if k is odd, so where...

0
Q: Q: Insurance company related probability problem

BrassicanI came across this problem: An insurance agent offers an insurance policy to 2n costumers. Each costumer accepts the offer with probability $0<\alpha<1$ and rejects it with probability $1-\alpha$. A costumer who accepted the offer choose randomly one of 3n service centers of the insurance compan...

0
Q: I need help with a Torricelli's theorem problem.

StefanThe Question: A full conical water tank of height 25m and diameter 50m drains in 100 minutes after the bottom plug (at the vertex) is removed. a) After the tank is drained, water is pumped into the tank at a rate of r=2m^3/min. Find the highest level to which the water will rise: Now I was able...

Welcome to Math.SE, Stefan. Words such as help are uninformative in titles. Please edit the title so that it better describes the specifics of your question. Do not hesitate to make it longer or include a formula if needed. This site uses MathJax formatting of formulas. More tips here. (autocomment)Normal Human 25 secs ago
 
10:36 AM
1
Q: How can you update a tag synonym description?

Jon SurrellThe map tag is a synonym of dictionary. This means map will be swapped for dictionary automatically. The problem is that the description displayed for the map tag is currently: a data structure relating keys to values, or a higher order function that applies a function to a list. For questio...

-1
Q: Kendo UI Numeric Textbox format binding

user3264425http://dojo.telerik.com/IZIBO I would like to bind the format of a Numeric textbox during the button click. Format is not changed during button click. Please let me know where i am going wrong.

 
0
Q: MCMC with dependent variables

banachraeumchenI want to run Metropolis-Hastings on a problem which involves two parameters that are not independent. I.e. I want to estimate both of these parameters. At the moment I'm trying to understand if this is possible and if so how or what to watch out for... I considered implementing a single-compone...

0
Q: Prove that: $ \forall x \in R_{+}{*}, arctanx> \frac{x}{1+x^2} $

TuanleeProve that: $ \forall x \in R_{+}{*}, arctanx> \frac{x}{1+x^2} $. I can't prove that. Help me please..

0
Q: How do I find the first four terms of the Taylor series expansion of $f(z)=\frac{1}{(z^2+ 1)}$

dragoonHow do I find the first four terms of the Taylor series expansion of $$f(z)=\frac{1}{(z^2+ 1)}$$ around $z=e^{i \pi/4}$

0
Q: What is the shape of the graph $|z-1|+|z+i|=2$ in the complex plane?

diyaWhat is the shape of the graph $|z-1|+|z+i|=2$ in the complex plane? $(A)\text{two points}\hspace{1cm}(B)\text{a line}\hspace{1cm}(C)\text{a parabola}\hspace{1cm}(D)\text{an ellipse}$ Let us take $z=x+iy$ $|(x-1)+iy|+|x+i(y+1)|=2$ $\sqrt{(x-1)^2+y^2}+\sqrt{x^2+(y+1)^2}=2$ Upon simplifying,$3x^2+3...

0
Q: Difference between sup and max

yokoI am working at some Fuzzy-Logic and I am having my problems with the inferece. While using the generalised modus ponens you are using this formula μB'(y) := sup{min(μA'(x),min(μA(x),μB(y))) | x∈X} for y∈Y My Question is, where is the Difference between the min/max Operators and the sup/in...

Consider adding a tag for a broader subject area to which the question belongs. Some of these tags might fit. (from a bot)Normal Human 23 secs ago
0
Q: Difficult sum involving Gamma functions and hypergeometric

CAFI have the following sum that I wish to evaluate: $$ \sum_{n=0}^{\infty} \left(\int_0^1 du\,u^{1+n+\epsilon}\right) \frac{\Gamma(n+2) \Gamma(1+\epsilon)}{\Gamma(n+3+\epsilon)} {}_2F_1\left(1, n+2, n+3+\epsilon, -\frac{a}{b}\right) = \sum_{n=0}^{\infty} \frac{1}{n+2+\epsilon} \frac{\Gamma(n+2) \Ga...

Words such as difficult are uninformative in titles. Please edit the title so that it better describes the specifics of your question. Do not hesitate to make it longer or include a formula if needed. More tips here. (autocomment)Normal Human 24 secs ago
0
Q: Maximization of a function subject to an inequality

user293051I have two constants $k,s$ and would like to find the maximum $t$ such that $$k^2 \frac{t(t-1)}{2}\left ( 1 - \frac{2}{(t+1)t} \right )^{st} < 1$$.

0
Q: Finding the co-efficient of A,B and C

MaikeleleFind the value of A,B and C given: 3x 2 + 4 ≅ A ( x + 2)2 + B (x + 2) + C I've managed to expand the brackets, however i am still confused on what to do next. Please help and explain the process.

This site uses MathJax formatting of formulas. More tips here. (autocomment)Normal Human 32 secs ago
0
Q: Compute the exact distribution of the MLE of a data conforming Poisson

Eric XuWe have observations ($y_i$,$x_i$) (i=1,...,n). Suppose that give $x_i$, $y_i$ is a discrete random variable with distribution $$ P(y_i=y|x_i=x) = \frac{e^{-x\beta}(x\beta)^y}{y!}, y=0,1,2,...$$ Assume that $x_i>0$ for all i. Compute the maximum likelihood estimator of $\beta$ and its exact dist...

Questions tend to get more attention when they have a tag for a broad area of mathematics relevant to the question. Some of these tags might fit. (from a bot)Normal Human 21 secs ago
0
Q: Linear Dependence In Other Vector Space

gboxI think it is trivial, but maybe a vector be linear dependence in one vector space but not in another vector space?

0
Q: Green's function for coupled ODEs or system of ODEs

Soumyajit RoyIs there any general formulation to find Green's functions of coupled ODEs or a system of ODEs? What is 'Green's function matrix'? Do Green's functions of coupled ODEs form a matrix which is called Green's function matrix? Please refer to a good text in this regard.

0
Q: Optimal grouping of squares

user293804I have a question from my professor and I need help on one part. The question reads: Suppose I have a $2n \times 2n$ grid of unit squares where $n>1$. Suppose we partition this grid into $2 \times 2$ contiguous grids of unit squares. We have a valid grouping of these squares if: At le...

0
Q: $\Bbb{R}^3 \ \Bbb{R}^2$ is disconnected. Show that $\Bbb{R}^4 \ \Bbb{R}^2$ is path connected?

SighMathSo far, I understand that we need to show that you can form a path, but I am very confused on how to go about it. Can anyone provide hints please?

0
Q: Guess the number of the prize & win the prize problem

M.E.There is prize in a box. The prize has a value of a positive integer between 1 and N and you are given N. To win the prize, you have to guess its value. Your goal is to do it in as few guesses as possible; however, among those guesses, you may only make at most g guesses that are too high. The v...

Title contains problem. Tagged proof-writing. Guess the number of the prize & win the prize problem
 
11:33 AM
0
Q: How to show that the function from the $\Bbb{S}^1$ to $\Bbb{R}$ is neither injective nor surjective.

NorrrrrrrrrrrrrrrrrrrI am aware that this is a continuous function however I am having trouble proceeding with a valid argument. Please help

 
0
Q: "Ignored Tags preferences" link under "Featured questions" is broken

Andrej Adamenko Go to Featured questions tab; Scroll to the bottom of the list of questions; Follow the link in the comment "Hiding questions because of your Ignored Tags preferences for tags:" Actual result: a new page opened with the following message: Page Not Found We couldn't find the page you request...

 
0
Q: Complex integration.

neelaWe can find complex integration of a function over a closed contour by residue theorem if there are only finite many singularity inside the contour. But my question how to find the integration if there are infinite many singularity inside the contour? Please help me solve this type of problem me...

Short title. Question contains please. Complex integration.
0
Q: Base 10 Decimal Expansion

lollercideProblem: If $x\in [0,1)$ write $a_1 = [10x], a_2=[100(x - \frac{a_1}{10})], a_3 = [1000(x - \frac{a_1}{10} - \frac{a_2}{100})], . . .$. Prove that $0 \leq a_k \leq 9$ for each $k$ and that $\sum_{n=1}^{\infty}\frac{a_n}{10^n}$ converges. Prove that $x = \sum_{n=1}^{\infty} \frac{a_n}{10^n}.$ Pro...

0
Q: Probability Deck of Cards

RSparkesIn a hand of 13 playing cards from a deck of 52 whats the probability of drawing exactly one king. My approach would be $${4 \choose 1}*{48 \choose 12}/{52 \choose 13}*{2}$$ I divided by 2 because I felt I had ordered the king and the other 12 cards chosen but this is wrong. Can someone please ...

Short title. Question contains please. Probability Deck of Cards
 
0
Q: Flag Offensive Post - No Evidence Found after Deletion

FabioI've encoutered some posts with strong evidence of Offensive content. I flag them as "Offensive" in the review process. One of them has been later "declined" because there were no evidence of offensive content on it. This is because the offensive post has been deleted in the time frame between th...

 
11:51 AM
0
Q: Prove or Disprove .

Shubham$\vDash \forall x (\alpha \lor \beta) \implies ( \forall x \alpha \lor \forall x \beta) $. I am not able to start this question, any help would be appreciated. Thanks in advance!

Short title. Prove or Disprove .
0
Q: How can we find the possible values of 'a' in a system of linear equestion?

Lap SinggWhat is/are the possible value(s) of 'a' such that the system of linear equestion in x,y and z has a unique solution? x-2y+az=0 y+3z=-1 -2x+3y+z=-2

-1
Q: the image set is dense

problemLet c be an irrational number and $f:~\mathbb{R}\to {{S}^{1}}\times {{S}^{1}}$         $t\to ({{e}^{2\pi it}},{{e}^{2\pi ict}}) $ Show that the image set $f(\mathbb{R})$ is dense in ${{S}^{1}}\times {{S}^{1}}$

0
Q: valuation of a particular element in $\Bbb{Z}_p$

Biswarup RayConsider $x \in \Bbb{Z}_p$. Then I want to find the valuation of $(1+p)^x-1$. I think that $val_p((1+p)^x-1)=1+val_p(x)$. Is this right?

0
Q: what does this sentence mean?

Marco Dinatsoli when the joint distribution of a set of items is different from the distributions of the individual items in the set. this is the context: Our discussion so far has focused on item-based top-N recommendation algo- rithms in which the recommendations were computed by taking into account rela...

0
Q: Proving the points $P,O,N$ are collinear

User 1upon0In a triangle $ABC$, let $M$ be the midpoint of side $BC$ and $N$ be the midpoint of median $AM$. Let $O$ be the circumcentre of triangle $ABM$. If the circumcircle of triangle $BOM$ cuts the side $AB$ at $P$(leaving the point B), prove that the points $P,O,N$ are collinear. I am quite weak at g...

0
Q: What is $x^T Ax$?

Mithlesh UpadhyayLet $A$ be the matrix $\begin{bmatrix}3 &1 \\ 1&2\end{bmatrix}$. What is the maximum value of $x^T Ax$ where the maximum is taken over all $x$ that are the unit eigenvectors of $A$? $5$ $\frac{(5 + \sqrt{5})}{2}$ $3$ $\frac{(5 - \sqrt{5})}{2}$ Eigenvalues of $A$ are $\frac{(5 \pm \sqrt{5})...

Short title. Question contains please. What is $x^T Ax$?
0
Q: Tricky Residue Integration

BrandonShow using residues that, for all $b > 0$ and $-1 < a < 0$, $$\int_0^{\infty}\frac{x^a \log x}{x+b}dx = \frac{\pi b^a}{\sin^2 \pi a}(\pi \cos \pi a - (\log b)(\sin \pi a))$$ I'm really confused on how many integrals in answer includes. I'm not sure if I need one for when b>0 and when a>-1 and wh...

Short title. Title contains tricky. Tricky Residue Integration
0
Q: How do I solve this normal distibution solving for a and b?

Matt.DoeX ~ norm(15, 1.25). I want to find a and b, such that P(a < X < b) = .95. I've attempted this multiple times and spent like two hours and all my answers are wrong. Please help!!!!

Short question. Question contains please. How do I solve this normal distibution solving for a and b?
0
Q: An strange operator in B(H)

Ali BagheriLet $H$ be a non-separable Hilbert space and $E$ be an orthonormal basis for $H$. Let $E_0$ be a countable subset of $E$ and $\{\delta_i\}_1^{\infty}$ be a bounded sequence of $(0,\infty)$. For given an arbitrary sequence $\{h_i\}_1^{\infty}$ of unit vectors in $H$, I am looking for an operator...

0
Q: Finding the similarity dimension of a variation of the Cantor Set.

LdpetkovIf we take the Cantor set and instead of removing the interval [1/3, 2/3], we remove the open interval [x,1-x], with 0

0
Q: How to formulate LP for shortest path problems?

mmsweI'm trying to understand how LP formulaton for shortest path problem. However I'm having trouble understanding constrains. Why this formulation work? http://ie.bilkent.edu.tr/~ie400/Lecture8.pdf I'm having trouble understanding how the constraints work at pages 15 and 17. I got the main idea a...

0
Q: DFA Containing A

technoI have L={Contains 'a'} and Alphabet(E)={a,b} Can i create a NFA Like this

Short title. Short question. DFA Containing A
0
Q: Proving that circumcenters are con-cyclic

anonymousI was completely lost when handed this at a math competition a couple of weeks ago. Please help. I am trying to understand how to do these questions so that I am ready for the next competition.

0
Q: Group of order $n=pq$

penguinaLet $G$ be a group of order n=pq, where $p$ and $q$ are prime numbers and let $x$ $\in$ $G$. My question is how hard is to compute $x^{-1}$ in $G$ ?

Short title. Short question. Group of order $n=pq$
0
Q: Are Morrey Spaces reflective?

van abelSince $L^{p,0}=L^p$ and $L^1$ is not reflective, thus in general Morrey space is not reflective, but how about for $L^{p,\lambda}$ with $1<p<+\infty$ and $0<\lambda<n$, where $n$ is the dimension of domain. What's more, it seems that the dual space for Morrey spaces are not clear so far?

0
Q: Is $\prod_{i=1}^{n}\frac{x_i}{\theta}=\frac{\prod_{i=1}^{n}x_i}{\theta}$?

LeafIs $$\prod_{i=1}^{n}\frac{x_i}{\theta}=\frac{\prod_{i=1}^{n}x_i}{\theta}$$ or $$\prod_{i=1}^{n}\frac{x_i}{\theta}=\frac{\prod_{i=1}^{n}x_i}{\prod_{i=1}^{n}\theta}=\frac{\prod_{i=1}^{n}x_i}{\theta^n}$$ ???

0
Q: There exist fractal with similarity dimension between 0 an 1?

LdpetkovHow to prove that there exist a fractal with similarity dimension D = x, where x is between 0 and 1?

0
Q: What is the importance of the axiomatization of set theory?

SediciosoWell I know that de axiomatization is important to establish certain laws, etc. But what other arguments do exist?

 
1:00 PM
0
Q: How about some comment features?

Jan TuroňAs I saw in answers to this question, many users (ab)uses comments to ping OP with apriori intention to delete their comment soon, often to leave a rather marginal note that is not completely meaningless. Many approved such behavior, but effectively it emulates volatile chat posts which are manua...

 
0
Q: Word describing a slice of multi-dimensional space

Aidan GomezI'm in the market for a mathematical (or otherwise) term to describe a slice of a hypercube. Tensor is out of the running as that's the name of the object I am slicing. The second I could use a hand with is a term to describe an index (or access point) that spans more than a single point in eac...

Consider adding a tag for a broader subject area to which the question belongs. Some of these tags might fit. (autocomment)Normal Human 20 secs ago
0
Q: Find the probability of are they siblings?

OngGiven there are 5 pairs of siblings, answer the question: What is the probability that 2 ppl randomly chosen are siblings? My answer: 10/10 + 1/9 What is the probability that all 4 people randomly chosen are all not related? i.e. no siblings among the 4 chosen. My answer: 1/10 + 1/9 + 1/8 + 1...

0
Q: Changing the order of a double integration

BalajiChange the order of integration in x varies from 0 to 2a and y varies from 0 to sqrt(2ax-x^2);integrand is (x^3 + y^2.x)/(sqrt(4.a^2.x^2 - (x^2 + y^2 )^2) dx dy and evaluate it. After changing the order, i found out the limits of integration as y varies from 0 to a and x varies from a-sqrt(a^2 ...

This site uses MathJax formatting of formulas. More tips here. (autocomment)Normal Human 20 secs ago
0
Q: Solution of a Functional equation

Radhakrishnamurty P What is the solution for the following functional equation? g(x).g(z) = g(x+z) + g(x-z) The solution given is: g(z) = 2cos(z). In the derivation of the result (using Taylor expansion), there is a step that is like this: g"(x) = b.g(x), where g"(x) is the second derivative of g(x) and b is a ...

This site uses MathJax formatting of formulas. More tips here. (from a bot)Normal Human 20 secs ago
0
Q: Use MCT on $(2x-4)^{-1/2}$

BCLCUse Monotone Convergence Theorem to prove that $$f(x) = [(2x-4)^{-1/2}]1_{(2,4]} + 01_{\{2\}}$$ is Lebesgue integrable on $[2,4]$. According to notes from a different class So I just replace $x$ w/ $f(x)$?

0
Q: if $F(x)=\ln{x}\ln{(1-x)}$ prove $ F'(x)>0$

 Kelantan Kotaan anyone please help me with the following proof: Let $$F(x)=\ln{x}\ln{(1-x)},0<x\le\dfrac{1}{2}$$ show that $$F'(x)>0$$ because $$F'(x)=\dfrac{(1-x)\ln{(1-x)}-x\ln{x}}{x(1-x)}$$ It suffices to show that $$G(x)=(1-x)\ln{(1-x)}-x\ln{x}>0,0<x\le\dfrac{1}{2}$$

0
Q: NFA Containing 01 as substring

technoGive the NFA accepting the language over the alphabet {O, 1} that have the set of strings which contain 01 as substring. I have created the following.Im i right?

0
Q: Rank Deficient matrix

tehseen fatimaThe rank of a matrix A is the dimension of the vector space generated by its columns. It shows the dependent and independent column of matrix A. Suppose I have a nxn matrix A with rank n/2. As a specific example, let A =[3 1 3 1;1 4 1 4;3 1 3 1;1 4 1 4] with the two eigenvalues 4.7639, 9.23361 ...

 
1:34 PM
0
Q: absolute minimum of function

Be HappyLet p and q be positive numbers satisfying 1/p+1/q= 1; and let f ∶ [0,+∞) → R be the function f(x) =1/p x^p− x +1/q Show that f has an absolute minimum at x = 1 and hence deduce the inequality ab ≤1/p a^p + 1/q b^q for any positive a and b. Hint. What can you say on the magnitude of p and q? I ...

This site uses MathJax formatting of formulas. More tips here. (autocomment)Normal Human 20 secs ago
0
Q: convergent sequence in Euclidean topology, exercise

JBIBBLet $X=(0,1)$ endowed with the Euclidean topology. Is the sequence $x_n=\frac{1}{n}$ convergent in the topological space X? I don't know how to start these kind of questions. Anybody who can give me an extensive answer? Thank you very much!

 
1:48 PM
0
Q: Find $sup_{x\in(0,+\infty)}(2^{-x}+2^{\frac{-1}{x}})$.

TuanleeFind $sup_{x\in(0,+\infty)}(2^{-x}+2^{\frac{-1}{x}})$. Help me please, I can't find it. I tried but it was difficult for me.

Short title. Short question. Question contains please. Find $sup_{x\in(0,+\infty)}(2^{-x}+2^{\frac{-1}{x}})$.
0
Q: Trouble with countability axioms

Michelle NaeemLet $X$ be a topological space having the property that every $ x \in X$ has some open neighborhood which is homeomorphic to some open subset of $\Bbb{R}^2$ . Give some justification or a counterexample for each of the following questions. (i) Is $X$ a first countable space? (ii) Is $X$ a second...

Title contains troubl. Question contains please. Trouble with countability axioms
0
Q: Finite eccentricity

SashaI define $P_t =\partial \{v<t\}$ a compact subset of $\mathbb{R}^n\setminus\{0\}$ where $v$ is a function defined on $\mathbb{R}^n\setminus\{0\}$ . I am looking at the evolution of the eccentricity $\Theta$ of these surfaces $P_t$. I want to justify why $\sup_t\Theta(P_t) < \infty$. Is this ob...

Short title. Finite eccentricity
 
-1
Q: Why very long suspension period?

Mithlesh UpadhyayMay I know why amWhy (1,019 helpful flags) got very long suspension period? The suspension period ends on Mar 21 '16 . I seen there he written very high voted answers.

 
00:00 - 14:0014:00 - 00:00

« first day (28 days earlier)      last day (546 days later) »